... ://7sage.com/lsat_explanations/lsat-46-section-3-question-24/
This sufficient assumption ... question really has me thrown. I' ... time with understanding the whole question.
Hey 7Sagers! Here's a question from a student I thought ... attempted negations for necessary assumption question Dec 2009, s3,q25. Thank ... ://7sage.com/lsat_explanations/lsat-59-section-3-question-25/
Clearly I'm having a hard time mapping out logic... Can someone please explain the correct answer? Also any tips on how to improve mapping out logic would be greatly appreciated!! Thanks!
Tough. Choice (E) is correct.
You can tell by using the
contrapositive on both of these statements:
If not (prices fall as rapidly as/more rapidly than
competitors),
then not (production costs fall as rapidly or more ...
... ://7sage.com/lsat_explanations/lsat-56-section-3-question-21
I understand Jon ... 's explanation on this question, but I was tricked by ... the question's word choice "revival" in ...
https://7sage.com/lsat_explanations/lsat-53-section-3-question-13
I can't see the difference between the flaw in B and the the flaw in the incorrect answer choices....
... ://7sage.com/lsat_explanations/lsat-34-section-3-question-20/
Hi guys, having ... of trouble with this weakening question. Here's my understanding of ...
... ://7sage.com/lsat_explanations/lsat-52-section-3-question-25/
Can someone please ... explain this question to me? It's the ... principle question and mentions distant periods etc ...
I cannot for the life of me figure out why C and D are wrong, can give a coherent detailed explanation for why they're wrong?
https://7sage.com/lsat_explanations/lsat-40-section-3-question-23/